LSAT and Law School Admissions Forum

Get expert LSAT preparation and law school admissions advice from PowerScore Test Preparation.

User avatar
 Dave Killoran
PowerScore Staff
  • PowerScore Staff
  • Posts: 5852
  • Joined: Mar 25, 2011
|
#59771
Complete Question Explanation
(The complete setup for this game can be found here: lsat/viewtopic.php?t=26553)

The correct answer choice is (A)

The question stem sets up the following sequence:


..... ..... ..... ..... W :longline: J


Although this rule interrelates the majors and nonmajors, more importantly it ties in the VFJ super-block. If W is assigned to a lower-number bench than J, it follows that W is assigned to a lower-numbered bench than V and F as well:

pt26_s98_g1_q6b.png
At this point, you can actually determine that answer choice (A) is correct because since W is assigned to a lower-numbered bench than J, only H remains to be assigned to the same bench as W, meaning that H must be assigned to a lower-numbered bench than F. However, if you do not see that inference immediately, move on to making hypotheticals.

Because the VFJ super-block is again forced into only two possible positions: benches 2 and 3 or benches 3 and 4, as in the previous two questions, you can quickly make the two applicable hypotheticals:

pt26_s98_g1_q6c.png

When the above hypotheticals are applied to the answer choices, it is apparent that answer choice (A) is correct. Answer choices (B), (C), and (E) could be true but do not have to be true. Answer choice (D) can never be true.
 EMD
  • Posts: 4
  • Joined: Jun 22, 2014
|
#15040
I was just reading through the answer explanations for Diagnostic Game 2, Chapter 4 of the Logic Games Bible. The question of issue is question 6. It's just not clear to me why the VFJ block can't be located at spaces 1/2. W could be placed above F.

Thanks so much!
User avatar
 Dave Killoran
PowerScore Staff
  • PowerScore Staff
  • Posts: 5852
  • Joined: Mar 25, 2011
|
#15041
Hi EMD,

Thanks for the question! Let's start by taking a look at the VFJ block again. The rule states that "Frank is assigned to the same bench as Victor." Consequently, W can't be assigned to the same bench as F. And, since question #6 states that "Wanda is assigned to a lower-numbered bench than is Joan," that then knocks the VFJ block out of 1-2.

Please let me know if that helps explain it. Thanks!
 DlarehAtsok
  • Posts: 50
  • Joined: Nov 18, 2015
|
#37671
Hi,

Do not you think that there is an easier explanation to this question (instead of working with hypotheticals)? Given that W is before J, it means that W cannot be with J, so it has to go with H, so A) follows directly?
User avatar
 Dave Killoran
PowerScore Staff
  • PowerScore Staff
  • Posts: 5852
  • Joined: Mar 25, 2011
|
#37749
Hi D,

Yes, that is the reasoning behind why (A) is correct, of course. You've shown the direct chain whereas I used hypos to show that since in my experience most people don't make that connection. But it's worth mentioning there in more pointed fashion.

Thanks!

Get the most out of your LSAT Prep Plus subscription.

Analyze and track your performance with our Testing and Analytics Package.